Consider the line shown on the graph.

Answers

Answer 1

The equation of the given line in the graph in slope intercept form is: y = 2x + 7

How to find the equation of a line?

The general formula for the equation of a line in slope intercept form is:

y = mx + b

where:

m is slope

b is y-intercept

From the given graph, the y-intercept is b = 7

Let us pick two coordinates which are:

(-3.5, 0) and (0, 7)

Thus:

(y - 0)/(x + 3.5) = (7 - 0)/(0 + 3.5)

y = 2(x + 3.5)

y = 2x + 7

Read more about Equation of Line at: https://brainly.com/question/18831322

#SPJ1


Related Questions

The table below shows the numbers of two to five bedroom houses in the Belmont Neighborhood. What is the mean number of bedrooms in a house in this neighborhood?
Number of bedrooms Frequency
2 7
3 35
4 56
5 27

Answers

The mean number of bedrooms in a house in this neighborhood will be: 3.824.

How to determine the mean number of bedrooms

In order to find the mean number of bedrooms in a house in the Belmont Neighborhood, we need to calculate the weighted mean by multiplying each number of bedrooms by their respective frequency.

Next, we will add the products, and then divide by the total frequency. So we can start as follows:

(2 * 7) + (3 * 35) + (4 * 56) + (5 * 27) = 14 + 105 + 224 + 135 = 478

Total frequency = 7 + 35 + 56 + 27 = 125

Therefore, the mean number of bedrooms = 478 / 125  approximately 3.824

Learn more about weighted mean here:

https://brainly.com/question/23902866

#SPJ1

What number increased by 3.5% of itself equals 621?

Answers

Answer:

Step-by-step explanation:

We need to find a number which increased by 3.5 of itself equals 621.

Let that number be x.

x increased by 3.5 of itself equals 621 implies x+ 3.5x = 621.

x(1+3.5) = 621

x(4.5) = 621

x = 621/4.5

x = 138

Therefore the required number is 138.

Answer: 138

Step-by-step explanation:

We need to find a number which increased by 3.5 of itself equals 621.

Let that number be x.

x increased by 3.5 of itself equals 621 implies x+ 3.5x = 621.

x(1+3.5) = 621

x(4.5) = 621

x = 621/4.5

x = 138

Therefore the required number is 138.

The wholesale cost of a dining room table is $1,315,80. The selling price of the table is $1,776.33. What is the markup for the table?
A 10%
B 35%
C 65%
D 90%

Answers

well, the markup is 1,776.33 - 1,315,80 = 461.33.

now, if we take 1315.80(origin amount), what is 461.33 off of it in percentage?

[tex]\begin{array}{ccll} Amount&\%\\ \cline{1-2} 1315.80 & 100\\ 461.33& x \end{array} \implies \cfrac{1315.80}{461.33}~~=~~\cfrac{100}{x} \\\\\\ 1315.8x=46133\implies x=\cfrac{46133}{1315.8}\implies x\approx 35[/tex]

I don’t know the answer I just need to answer a question so I can ask one sorry

Matt can save $225 per month that he puts into a savings
account earning 5% annual interest. How much will he have
saved after 2 years?

Answers

Answer:

FV ≈ $5,673.56

Step-by-step explanation:

To calculate the total amount that Matt will have saved after 2 years of saving $225 per month at an annual interest rate of 5%, we can use the formula for the future value of an annuity:

FV = P * (((1 + r/12)^(n*12) - 1) / (r/12))

where:

FV is the future value of the annuity

P is the periodic payment (in this case, $225 per month)

r is the interest rate per year (in this case, 5%)

n is the number of years (in this case, 2)

Substituting the given values, we get:

FV = $225 * (((1 + 0.05/12)^(2*12) - 1) / (0.05/12))

Using a calculator, we get:

FV ≈ $5,673.56

Therefore, after 2 years of saving $225 per month at an annual interest rate of 5%, Matt will have saved approximately $5,673.56.

Bokamoso rides a bicycle to school every day (taking the same route). If she rides at 16 km/h, it takes her 45 minutes to get to school. 1. How far does Bokamoso ride on the way to school?​

Answers

5.40 km is the distance Bokamoso ride on the way to school

What is speed, distance and time?

Speed is determined by dividing the distance travelled by the time taken to travel it. It gives the amount of time it took to travel a certain distance divided by the distance travelled.

Speed is inversely correlated with time and directly correlated with distance. As a result, distance equals speed times time, and time equals distance divided by speed; as speed rises, distance travelled will shorten, and vice versa.

Bokamosa rides at 16 km / hr

To get to school = 45 min

= 45 * 0.75 hour

= 0.3375 hr

Distance = Speed * Time

d = 16 * 0.3375

= 5.40 km

Hence, 5.40 km is the distance Bokamoso ride on the way to school.

To know more about distance check the below link:

https://brainly.com/question/17273444

#SPJ9

f(x) = x2 + 4; interval [0, 5]; n = 5; use left endpoints

Answers

Therefore, the Left Riemann Sum for this function, interval, and number of subintervals is 50.

The left endpoint rule is what?

The top-left corner of these rectangles touched the y=f(x) curve. In other words, the value of f at the subinterval's left endpoint determined the height of the rectangle over that subinterval. This technique is called the left-endpoint estimate because of this.

With left endpoints and n = 5 subintervals, we may use the Left Riemann Sum formula to approximate the area under the curve of f(x) = x2 + 4 over the range [0, 5]:

Left Riemann Sum = ∑[i=1 to n] f(x_i-1) Δx

In this case, a = 0, b = 5, n = 5, and we will use the left endpoints, so:

Δx = (5 - 0)/5 = 1

Using the left endpoints, the subintervals and their left endpoints are:

[0,1],[1,2],[2,3],[3,4],[4,5]

[tex]so,\; x_0 = 0, x_1 = 1, x_2 = 2, x_3 = 3, x_4 = 4.[/tex]

Now we can calculate the Left Riemann Sum:

Left Riemann Sum= [tex]f(x_0)\Delta x + f(x_1)\Deltax + f(x_2)\Deltax + f(x_3)\Deltax + f(x_4)\Deltax[/tex]

= f(0)×1+f(1)×1+f(2)×1+f(3)×1 + f(4)×1

= 4+5+8+13+20

= 50

To know more about the function visit:-

brainly.com/question/12431044

#SPJ1

Find the Volume of 10 ⅕in 4 ¼in 2in

Answers

The volume of the rectangular prism is (867/20) cubic inches.

What is volume?

A volume is just the amount of space taken up by any three-dimensional solid. A cube, a cuboid, a cone, a cylinder, or a sphere are examples of solids. Volumes differ depending on the shape.

To find the volume of a rectangular prism, we need to multiply its length, width, and height. In this case, the length is 10 1/5 inches, the width is 4 1/4 inches, and the height is 2 inches. We can convert the mixed numbers to improper fractions and then multiply:

length = 10 1/5 inches = (51/5) inches

width = 4 1/4 inches = (17/4) inches

height = 2 inches

Volume = length x width x height

= (51/5) inches x (17/4) inches x 2 inches

= (867/20) cubic inches

Therefore, the volume of the rectangular prism is (867/20) cubic inches.

Learn more about volume on:

https://brainly.com/question/6204273

#SPJ1

Is a measure of 30 inches​ "far away" from a mean of 15 ​inches? As someone with knowledge of​ statistics, you answer​ "it depends" and request the standard deviation of the underlying data.
​(a) Suppose the data come from a sample whose standard deviation is 3 inches. How many standard deviations is inches from 15 ​inches?
​(b) Is 30 inches far away from a mean of 15 ​inches?
​(c) Suppose the standard deviation of the underlying data is 10 inches. Is 30 inches far away from a mean of 15 ​inches?

Answers

Hence, 30 inches deviate by 1.5 standard deviations from the 15-inch mean as the underlying data's standard deviation is 10 inches.

what is standard deviation ?

The standard deviation of statistics is a measurement of how much a group of data values deviate from their mean (average) value. While a significant standard deviation suggests that the pieces of information are dispersed throughout a broader range of values, a low standard deviation suggests that the data points are generally close to the mean.

given

a) If the sample's standard deviation is 3 inches, then the number of standard deviations from the mean of 15 inches that are 30 inches is:

z = (30 - 15) / 3 = 5

b) The context of the data and the standard deviation will determine whether 30 inches is distant from a mean of 15 inches.

A departure of 15 inches from the mean, meanwhile, can be viewed as being significantly off the mean if the standard deviation is minimal.

c) If the underlying data's standard deviation is 10 inches, then the number of standard deviations from the mean of 15 inches that are 30 inches is:

z = (30 - 15) / 10 = 1.5

Hence, 30 inches deviate by 1.5 standard deviations from the 15-inch mean as the underlying data's standard deviation is 10 inches.

To know more about standard deviation visit:

https://brainly.com/question/23907081

#SPJ1

Use the information below to determine the new coordinates of the image under the given translation.
Triangle ABC with vertices
A(0, 7), B(7, 3),
and C(1, 4): (x, y) → (x − 3, y - 4)

Answers

The new triangle A'B'C' after the translation is A'(-3, 3), B'(4, -1), and C'(-2, 0).

What is translation?

A shape is translated when it is moved up, down, left or right without turning. They are congruent if the translated shapes (or the image) seem to be the same size as the original shapes. They have only changed their direction or directions.

To perform the given translation (x, y) → (x − 3, y - 4) on the triangle ABC with vertices A(0, 7), B(7, 3), and C(1, 4), we need to apply the same transformation to each vertex of the triangle and obtain their new coordinates.

For vertex A(0, 7), the transformation gives:

(x, y) → (x − 3, y - 4)

(0, 7) → (0 - 3, 7 - 4)

The new coordinates of A are (-3, 3).

For vertex B(7, 3), the transformation gives:

(x, y) → (x − 3, y - 4)

(7, 3) → (7 - 3, 3 - 4)

The new coordinates of B are (4, -1).

For vertex C(1, 4), the transformation gives:

(x, y) → (x − 3, y - 4)

(1, 4) → (1 - 3, 4 - 4)

The new coordinates of C are (-2, 0).

Therefore, the new triangle A'B'C' after the translation is A'(-3, 3), B'(4, -1), and C'(-2, 0).

Learn more about translation on:

https://brainly.com/question/1046778

#SPJ1

Which expression represents the phrase The sum of w and twenty

A. 20w
B. w + 20
C. w^20
D. 20 x w

Answers

I believe that would be A. similar to combining like terms, but because it does not have another number to add, it would give the variable at the end.

The histograms display the frequency of temperatures in two different locations in a 30-day period.

A graph with the x-axis labeled Temperature in Degrees, with intervals 60 to 69, 70 to 79, 80 to 89, 90 to 99, 100 to 109, 110 to 119. The y-axis is labeled Frequency and begins at 0 with tick marks every one unit up to 14. A shaded bar stops at 10 above 60 to 69, at 9 above 70 to 79, at 5 above 80 to 89, at 4 above 90 to 99, and at 2 above 100 to 109. There is no shaded bar above 110 to 119. The graph is titled Temps in Sunny Town.

A graph with the x-axis labeled Temperature in Degrees, with intervals 60 to 69, 70 to 79, 80 to 89, 90 to 99, 100 to 109, 110 to 119. The y-axis is labeled Frequency and begins at 0 with tick marks every one unit up to 14. There is no shaded bar above 60 to 69. A shaded bar stops at 4 above 70 to 79, at 4 above 80 to 89, at 6 above 90 to 99, at 6 above 100 to 109 and at 10 above 110 to 119. The graph is titled Temps in Beach Town.

When comparing the data, which measure of center should be used to determine which location typically has the cooler temperature?

Median, because Sunny Town is symmetric
Mean, because Sunny Town is skewed
Median, because Beach Town is skewed
Mean, because Beach Town is symmetric

Answers

Answer:

Whats the question?

Step-by-step explanation:

5. center (-4,-7), tangent to x = 2

Answers

The equation of the circle is therefore: [tex](x + 4)^2 + (y + 7)^2 = 36[/tex]

What is circle?

A circle is a two-dimensional geometric shape that consists of all the points that are equidistant from a single fixed point called the center.

To find the equation of a circle with center (h, k) and radius r, the general equation is:

[tex](x - h)^2 + (y - k)^2 = r^2[/tex]

In this case, the center of the circle is (-4, -7) and the circle is tangent to the vertical line x = 2. This means that the radius of the circle is the distance from its center to the line x = 2, which is 6 units.

The equation of the circle is therefore:

[tex](x - (-4))^2 + (y - (-7))^2 = 6^2\\\\(x + 4)^2 + (y + 7)^2 = 36[/tex]

Alternatively, we can also find the equation of the circle by using the fact that the line passing through the center of the circle and the point of tangency is perpendicular to the tangent line. Since the tangent line x = 2 is vertical, the line passing through the center (-4,-7) and the point of tangency must be horizontal, and its equation is y = -7.

The distance from the center (-4,-7) to the point of tangency on the line x = 2 is the radius of the circle, which is 6 units. The x-coordinate of the point of tangency is 2, so the distance from (-4,-7) to (2,-7) is 6 units. Therefore, the center of the circle (-4,-7) is 6 units away from the point (2,-7) along a horizontal line.

The equation of the circle is therefore:

[tex](x - (-4))^2 + (y - (-7))^2 = 6^2\\\\(x + 4)^2 + (y + 7)^2 = 36[/tex]

To learn more about circle visit:

https://brainly.com/question/24375372

#SPJ1

Please help quick 55 points giving away!

Answers

The correct choice is (4) x² = 34² +29² - 2(34)(29)*cos(56).

What do you Mean by Trigonometry?

Trigonometry is one of the branches of mathematics that deals with the relationship between the sides of a triangle (a right-angled triangle) and its angles. There are six types of trigonometry.

We can use the laws of cosines and sines to solve this problem.

According to the law of cosines, we have:

cos(B) = (a² c² - b²) / 2ac

where a, b and c are the  lengths of opposite sides of angles A, B and C. In this case, we get that a = 34, b = x, and c = 29, and we know that angle C is 56 degrees . Because:

cos(B) = (34² 29² - x²) / (23429)

According to the Law of Sines, we have:

sin(B) / b = sin(C) / c

Substituting the given values ​​we get:

sin(B) / x = sin(56) / 29

Solving Sin(B) we get:

sin(B) = x * sin(56) / 29

Now we can use the identity sin²(B) cos²(B) = 1 to eliminate sin(B) and solve for x:

(x * sin(56) / 29)² cos²(B) = 1

Expanding and substituting  cos(B) from the previous expression, we get:

(x² * sin²(56) / 29²) ((34² 29² - x²) / (2(34)(29))²= 1

Simplifying and solving for x², we get:

x² = 34²+ 29² - 2(34)(29)*cos(56)

Therefore, the correct choice is (4) x² = 34² +29² - 2(34)(29)*cos(56).

Learn more about Six types of trigonometry here

https://brainly.com/question/30283045

#SPJ1

If you horizontally…

Answers

Check the picture below.

so following the template below, a shift 2 units to the left, means C=2

[tex]G(x)=\sqrt{x+2}[/tex]

x^2+y^2+6x+2y-6=0 what is the center of this circle? what is the radius of this circle?

Answers

Answer:

center : (-3, -1) radius : 4

Find each value or measure.

x =

(40 points) will give brainiest for effort

Answers

Answer:

x = 5

Step-by-step explanation:

given FH is a diameter , then

arc FH = 180°, that is

FG + GH = 180°

103° + GH = 180° ( subtract 103° from both sides )

GH = 77°

the central angle HIG is equal to the arc that subtends it GH , so

∠ HIG = 77°

∠ EIF and ∠ HIG are vertically opposite angles and are congruent , then

∠ EIF = 77°

the arc EF subtends the central angle EIF , so

EF = 77°

the sum of the 3 arcs = 180° , that is

HD + DE + EF = 180

15 + 18x - 2 + 77 = 180

18x + 90 = 180 ( subtract 90 from both sides )

18x = 90 ( divide both sides by 18 )

x = 5

then arc DE = 18x - 2 = 18(5) - 2 = 90 - 2 = 88°

Find the critical value t

Answers

The answer of the given question based on the Critical value is , , the critical value to for the confidence level c = 0.99 and sample size n = 22 is 2.819.

What is Critical value?

In statistics, the critical value is a value that is used to determine whether to reject the null hypothesis in a hypothesis test. It is based on the chosen level of significance, which is the maximum probability of making a Type I error (rejecting a true null hypothesis). The critical value is determined by the sampling distribution of the test statistic, which is often a t-statistic or z-statistic, depending on the test and the characteristics of the population being studied.

To find the critical value t for a 99% confidence level and a sample size of 22, we need to use a t-distribution table or a calculator.

Using a t-distribution table with 21 degrees of freedom (n-1), we find that the critical value for a 99% confidence level is approximately 2.819.

Therefore, critical value for confidence level c = 0.99 and sample size n = 22 is 2.819 (rounded to nearest thousandth).

To know more about Null hypothesis visit:

https://brainly.com/question/30404845

#SPJ1

There is a 25% chance that a vowel is drawn from a bag of random letter tiles. what is the probability of drawing a vowel, placing it back in the bag, and then drawing a consonant

Answers

The probability of drawing a vowel, placing it back in the bag, and then drawing a consonant is 0.1875 or 18.75%.


Calculating the probability

The probability of drawing a vowel from the bag of random letter tiles is 25%. Since the tile is replaced after drawing, the probability of drawing a vowel on the second draw is also 25%.

The probability of drawing a vowel and then a consonant can be calculated by multiplying the probabilities of each event:

P(vowel and consonant) = P(vowel) x P(consonant)

= 0.25 x 0.75

= 0.1875

Read more about probability at

https://brainly.com/question/251701

#SPJ1

the temperature at 8 a.m. was -8.7 F. At 1 p.m. it was 9.3 F. What integer represents the change in the temperature from morning to afternoon?

Answers

The integer that represents the change in temperature from morning to afternoon is 18.

What is integer?

The group of counting numbers that can be written without a fractional component includes zero and both positive and negative integers. An integer can, as was already established, be either positive, negative, or zero.

To find the change in temperature from morning to afternoon, we need to subtract the morning temperature from the afternoon temperature:

9.3 F - (-8.7 F) = 9.3 F + 8.7 F = 18 F

Therefore, the integer that represents the change in temperature from morning to afternoon is 18.

Learn more about integers on:

https://brainly.com/question/28281557

#SPJ9

Chef Lori uses 1 1/4 cup of milk for every 3/4 cup of broth in her soup recipe. This recipe will feed 4 people. Lori is making soup for a banquet that will feed 120 people. How many gallons of milk and broth combined does she need.​

Answers

Answer: To make soup for 4 people, Lori needs 1 1/4 cups of milk for every 3/4 cup of broth. Therefore, the ratio of milk to broth is:

(1 1/4 cups milk) / (3/4 cup broth) = (5/4 cups milk) / (3/4 cups broth) = (5/3) cups milk per cup of broth

To make soup for 120 people, Lori needs to multiply the ratio by 30 (120/4) to get the total amount of milk and broth needed:

(5/3 cups milk per cup of broth) * 30 = 50 cups of milk per 30 cups of broth

To convert cups to gallons, we divide by 16 (since there are 16 cups in a gallon):

50/16 = 3.125 gallons of milk

30/16 = 1.875 gallons of broth

Therefore, Lori needs a total of 3.125 + 1.875 = 5 gallons of milk and broth combined.

Step-by-step explanation:

Using a table, find the range of the function for the given domain:

f(x)=2x+7 with domain: x = {2, 3, 5, 9}



A. y = {9, 10, 12, 16}
B. y = {11, 13, 17, 25}
C. y = {4, 6, 10, 18}
D. y = {-11, -13, -17, -25}

Answers

Given;

f(x)=2x+7

domain: x = {2, 3, 5, 9}

Explanation:

Replace each x value and solve for y

x=2

f(2)= 2(2) + 7 = 4 + 7 = 11

x=3

f(3)= 2(3) + 7 = 6 + 7 = 13

x=5

f(5)= 2(5) + 7 = 10 + 7 = 17

x=9

f(9)= 2(9) + 7 = 18 + 7 = 25

Answer:B. y = {11, 13, 17, 25}

We can find the range of the function by plugging in each value in the domain into the function and listing the output values:

When x = 2, f(2) = 2(2) + 7 = 11

When x = 3, f(3) = 2(3) + 7 = 13

When x = 5, f(5) = 2(5) + 7 = 17

When x = 9, f(9) = 2(9) + 7 = 25

Therefore, the range of the function for the given domain is y = {11, 13, 17, 25}.

Hence, the correct answer is B. y = {11, 13, 17, 25}.

1.) Write a48 as a product of two powers with the same
base in four different ways. Only use positive exponents.

Answers

The given exponents term can be written in the form of [tex](a^{4})^{12}[/tex] .

What about exponents?

Exponents are a mathematical operation that involves multiplying a base number by itself a certain number of times. Exponents are often referred to as powers, and the base number is raised to a power, which is usually represented by a superscript number. For example, 2^3 means 2 raised to the power of 3, which is 2 x 2 x 2 = 8.

Exponents are commonly used in many areas of mathematics, including algebra, calculus, and geometry. They are used to represent very large or very small numbers, to simplify calculations, and to express repeated multiplication in a more compact form. Exponents can also be negative or fractional, allowing for the representation of roots and other mathematical concepts.

According to the given information:

In the given question [tex](a^{48} )[/tex] can be written as,

[tex](a^{4})^{12}[/tex] = [tex](a^{4})^{12} = a^{48}[/tex]

To know more about exponent visit:

https://brainly.com/question/16998279

#SPJ1

Given the figure below .find X and Y to three significant digits.Write your answer in the answer box provided below

Answers

Check the picture below.

Make sure your calculator is in Degree mode.

[tex]\cos(25^o )=\cfrac{\stackrel{adjacent}{x}}{\underset{hypotenuse}{12}}\implies 12\cos(25^o)=x\implies \boxed{10.876\approx x} \\\\[-0.35em] ~\dotfill\\\\ \sin(25^o )=\cfrac{\stackrel{opposite}{z}}{\underset{hypotenuse}{12}}\implies 12\sin(25^o)=z \\\\[-0.35em] ~\dotfill\\\\ \sin(50^o )=\cfrac{\stackrel{opposite}{z}}{\underset{hypotenuse}{y}}\implies y=\cfrac{z}{\sin(50^o)}\implies y=\cfrac{12\sin(25^o)}{\sin(50^o)}\implies \boxed{y\approx 6.62}[/tex]

Ian has a deck that measures 20 feet by 10 feet. He wants to increase each dimension by equal lengths so that its area is tripled. By how much should he increase each dimension?

Answers

Answer:

10 feet

Step-by-step explanation:

Let's start by finding the current area of the deck:

Area = length x width = 20 ft x 10 ft = 200 sq ft

If Ian increases each dimension by the same amount, let's call this amount "x", then the new dimensions of the deck will be:

Length = 20 ft + x

Width = 10 ft + x

The new area of the deck will be:

New Area = (20 ft + x) x (10 ft + x)

We know that Ian wants the new area to be triple the original area of 200 sq ft, so:

New Area = 3 x 200 sq ft

New Area = 600 sq ft

Substituting this into the equation above and solving for "x", we get:

(20 ft + x) x (10 ft + x) = 600 sq ft

200 + 30x + x^2 = 600

x^2 + 30x - 400 = 0

(x + 40) (x - 10) = 0

We discard the negative solution, and we get:

x = 10 ft

Therefore, Ian should increase each dimension by 10 feet to triple the area of his deck.

To confirm, we can check the original area of the deck which is 20 ft x 10 ft = 200 sq ft.

If Ian increases each dimension by 10 feet, the new dimensions of the deck will be 30 ft x 20 ft. The new area of the deck will be 30 ft x 20 ft = 600 sq ft. This is triple the original area of the deck (200 sq ft), which is what we wanted to achieve.

Therefore, increasing each dimension by 10 feet will triple the area of the deck as required.

Ian should increase both the sides by 10 feet each.

This is a simple mathematics problems related to the topic of mensuration.

Firstly we calculate the current area of the deck:

Area of the deck (rectangle) = l x b

Area of the deck (rectangle) = 20 x 10

Area of the deck (rectangle) =  200 square feet

Since Ian wants to triple the area of the deck, the required area is 600 square feet.

We now look at pairs of numbers multiplying which will give us 600:

(1,600) (2,300) (3,200) (4,150) (5,120) (6,100) (8,75) (10,60) (12,50) (15,40) (20,30) (25,24)

Out of the above pairs only one pair fulfils Ian's criteria to increase the length and breadth of the deck by equal measure, i.e., (20,30). So, Ian should increase the length and breadth of his deck by 10 feet each.

To know more about Mensuration,

https://brainly.com/question/14269400

Working with Actual Interest Earned
Debrorah puts $300 in a CD that earns 3.0% APR, compounded
quarterly, for 1 year. She is taxed at a rate of 20% on the interest she
earns. She takes her money out after 9 months and pays the penalty
fee of 1 quarter's interest.
Deposit
PENALTY
7. What is the total amount of interest that Deborah earns, after taxes?
Type answer here.

Answers

The account has an annual interest rate of 6.80%, compounded continuously.

What is interst rate?

Interest is the cost associated with the boring money or to return on for lending money it is typically expressed as a percent of the principal amount borrowed or lent and usually the paid periodically over the life of loan interest can also be on from investment such as saving account bonds and the other financial instrument.

The interest rate of the account can be calculated as follows:

Rate = ln(1,698.49/1,051)/(8×ln(1))

Rate = 0.0680 or 6.80%

This means that the account has an annual interest rate of 6.80%, compounded continuously.

To know more about intrest rate click-
https://brainly.com/question/16134508
#SPJ1

5.96 x 10 6 - 7.29 x 10 5

Answers

Answer:-133.69

Step-by-step explanation:

5.96 x 106 -7.29 x 105

631.76 -7.29 x 105

631.76 - 765.45

= -133.69

What is the price per cubic inch for the regular size popcorn

Answers

the price per cubic inch for the regular size popcorn is #29.85

How to determine the price?

You should understand that Price is what a buyer pays to acquire products from a seller.  It takes into account the overall value of the offering, including the value of all raw materials and service that went into making an offering, as well as its markup or margin

The popcorn has 6 faces and the area of the popcorn is given as

Area = LBH

Length

Bredth

Height

Area= 3*5*6 = 90 cm²

Therefore, Price is given as $1.99

This implies that 1.99*90 = $179.1

Therefore the price per inch is 179.1/6 = #29.85

Learn more about price on https://brainly.com/question/19091385

#SPJ1

You invest 2,500 in a stock plan, buying 125 shares. If each share increases in value by 10%, how much is each share worth

Answers

Answer:

Step-by-step explanation:

answer 1,250

PLEASE HELP SOLVE MATH

Answers

The equation of the form y = c + b logₐX is y = -3 + 3 Log₃X

How to derive the equation?

Recall that Slope-intercept form of a linear equation is where one side contains just “y”. It looks like y = mx + “b” where “m” and “b” are numbers. This form of the equation is very useful because the coefficient of "x" (the "m" value) is the slope of the line and the constant (the "b" value) is the y-intercept at (0, b)

The equation of the line is give as

y-y₁ = m(x-x₁) + c

Where m is the slope and c is the intercept

This implies that

y -3 = m(x -2)

But slope is given as  m = (y₂ - y₁)/(x₂-x₁)

m = (9-3)/ 4-2) = 6/2 = 3

Then, the equation is

y - 3 = 3x - 6

Collecting like terms we have

y = 3x -6+3

y = 3x -3

Writing this in the form y = c + b logₐX

y = -3 + 3 Log₃X

Learn more about slope intercept form on https://brainly.com/question/29146348

#SPJ1

The equation of the function in the form y = c + b log_a(x) is y = logₓ2

Calculating the equation of the function

We can start by assuming that the equation is of the form y = c + b log_a(x), where c and b are constants and a is the base of the logarithm.

Using the point (2, 3), we get:

3 = c + b log_a(2)

Using the point (4, 9), we get:

9 = c + b log_a(4)

Simplifying the second equation using the logarithmic identity

loga(4) = 2 loga(2), we get:

3 +  2b loga(2) = 9

Substituting the first equation into this one, we get:

3 = 9 - 2b loga(2)

So, we have

-6 = - 2b loga(2)

Divide

bloga(2) = 3

So, we have

b = 3 / log_a(2)

Substituting this value of b into the first equation, we get:

3 = c + b log_a(2)

3 = c + 3 / log_a(2) * log_a(2)

So, we have

c = 0

Therefore, the equation of the curve is y = (2 / log_a(2)) log_a(x)

We can simplify this equation by using the logarithmic identity log_a(x^b) = b log_a(x):

y = 2 log_a(x) / log_a(2)

y = (2 / log(2)) log(x)

So the final equation is:

y = logₓ2

Read more about logarithm at

https://brainly.com/question/28041634

#SPJ1

What are the answers to these questions?
absolute maximum is ? and it occurs at x = ?
absolute minimum is ? and it occurs at x = ?

Answers

The absolute maximum value of f(x) is 4.5 and it occurs at x = -0.5, and the absolute minimum value of f(x) is -0.241 and it occurs at x = 0.833

Calculating the absolute maximum and minumum of the function

To find the absolute maximum and minimum of the function f(x) = 4x^3 - 2x^2 - 5x + 3, we need to take the derivative of the function and set it equal to zero to find the critical points.

f(x) = 4x^3 - 2x^2 - 5x + 3

f'(x) = 12x^2 - 4x - 5

Setting f'(x) = 0, we get:

12x^2 - 4x - 5 = 0

Using the quadratic formula, we get:

x = -0.5 or x = 0.833

Now we need to check the values of f(x) at the critical points and the endpoints of the interval

f(-0.5) = 4(-0.5)^3 - 2(-0.5)^2 - 5(-0.5) + 3 = 4.5

f(0.833) = 4(0.833)^3 - 2(0.833)^2 - 5(0.833) + 3 = -0.241

So the absolute maximum it occurs at x = -0.5, and the absolute minimum occurs at x = 0.833

Read more about absolute maximum at

https://brainly.com/question/31400719

#SPJ1

Other Questions
Which sentence from the passage is a major supporting detail a patient is diagnosed with heart failure (hf), and the prescriber has ordered digoxin. the patient asks what lifestyle changes will help in the management of this condition. the nurse will recommend which changes? J.P. Morgan was one of the wealthiest Americans ever. How did he gain much of his early wealth? a.He was born poor but built an empire by running a successful steel business. b.He invented the type of electricity we use in our homes today. c.He bought and sold railroad stocks. d.He inherited most of his wealth from his father. PLEASE ANSWER (ill give brainliest)Which feature of the model represents a transfer of energy? A. The sticks connecting the balls that represent atoms B. The balls that represent atoms C. The arrow between the reactants and the products D. The plus sign (+) between sugar and oxygen I need help with this question can you help? a particle with a cahrge of 1 c is moving at 45 angle with respect to the positive x axis in teh horizontal xy-plane. the velocity of the charge is 1 m/s. a magnetic field of 1 t is directed in the negative x direction. what is the magnetic force acting on the charge? In order to estimate the population size of the woodlice in her garden, Jessica used the mark-recapture method: she trapped, marked and released 20 woodlice. Three days later, she trapped 15 woodlice and observed that 10 had marks. Use the following formula to estimate the woodlice population size: population size = 1st sample 2nd sample 2nd sample that had been marked.NEED HELP ASAP 100 PTS!!!!!!! Select whether the situation could yield variable data, and if so, select the option with the best statistical question. The town council members want to know how much recyclable trash a typical household in town generates each week. A. Variable data. How many members are on the council?B. Variable data. When are the recyclables collected?C. Not variable. D. Variable data. How many pounds of recyclables trash does each household generate? carbon tetrachloride displays a triple point at and a melting point (at ) of . which state of carbon tetrachloride is more dense, the solid or the liquid? explain. Ned collected rocks at the beach. He placed 12 ounces of rocks in each of 4 different bags. How many ounces of rocks Ned collect? perhaps the most significant turning point of the european war was the The surface area of a prism can be determined by the formula S = 2B + Ph, where B isthe area of the base, P is the perimeter of the base and h is the height of the prism. Howcan this equation be rearranged to solve for the area of the base, B? Your broker charges $0.0020 per share per trade. The exchange charges $0.0119 per share per trade for removing liquidity and credits $0.0101 per share per trade for adding liquidity. The current best BID price for stock XYZ is $72.81 per share, while the current best ASK price is $72.82 per share. You post an order to buy XYZ at the current best BID price and wait. Shortly after, the best BID and ASK prices move lower (down) by one cent each. Your buy order is executed. Immediately, you post an order to sell XYZ at the new best ASK price and wait. Shortly after, the best BID and ASK prices move higher (up) by one cent each. Your sell order is executed. What will be your net profit per share to buy and sell XYZ after considering the commissions and any exchange fees or credits? $0.0150 $0.0154 $0.0158 $0.0162 $0.0166 Newton's first law of motion states that an object in motion will stay in motion unless acted upon by a force.What are two forces on earth that could change the motion of the object?Responsesfriction and gravityfriction and gravityspeed and accelerationheat and lightdirection and time Reasons Why immigrants are in Khayelitsha areas ? Find the area? For this shape pleae Solve for XPlease show step by step(X - 4)^2 = 25 Assume the last dividend was $2.00 and investors require 9% toinvest in this companys stock. What if g = 0% for 3 years beforelong-run growth of 4%please inlcude work if reynolds experiment was performed with a 38-mm- id pipe, what flow velocity would occur at transition? 5. to make a group of monochromatic colors, you simply have to add white or black to one color. true false